2022 AMC 12A Problems/Problem 14

Revision as of 19:32, 11 November 2022 by Inventivedant (talk | contribs) (Created page with "==Problem== What is the value of <cmath>(\log 5)^{3}+(\log 20)^{3}+(\log 8)(\log 0.25)</cmath> where <math>\log</math> denotes the base-ten logarithm? <math>\textbf{(A)}~\dis...")
(diff) ← Older revision | Latest revision (diff) | Newer revision → (diff)

Problem

What is the value of \[(\log 5)^{3}+(\log 20)^{3}+(\log 8)(\log 0.25)\] where $\log$ denotes the base-ten logarithm?

$\textbf{(A)}~\displaystyle\frac{3}{2}\qquad\textbf{(B)}~\displaystyle\frac{7}{4}\qquad\textbf{(C)}~2\qquad\textbf{(D)}~\displaystyle\frac{9}{4}\qquad\textbf{(E)}~\displaystyle\frac{5}{2}$

Solution

Let $\text{log } 2 = x$. The expression then becomes \[(1+x)^3+(1-x)^3+(3x)(-2x)=\boxed{2}.\]

-bluelinfish

See Also

2022 AMC 12A (ProblemsAnswer KeyResources)
Preceded by
Problem 13
Followed by
Problem 15
1 2 3 4 5 6 7 8 9 10 11 12 13 14 15 16 17 18 19 20 21 22 23 24 25
All AMC 12 Problems and Solutions

The problems on this page are copyrighted by the Mathematical Association of America's American Mathematics Competitions. AMC logo.png